Difference between revisions of "2024 AMC 12B Problems/Problem 19"
(12 intermediate revisions by 5 users not shown) | |||
Line 11: | Line 11: | ||
<math>\textbf{(A)}~\frac{3}{4}\qquad\textbf{(B)}~\frac{5\sqrt{3}}{11}\qquad\textbf{(C)}~\frac{4}{5}\qquad\textbf{(D)}~\frac{11}{13}\qquad\textbf{(E)}~\frac{7\sqrt{3}}{13}</math> | <math>\textbf{(A)}~\frac{3}{4}\qquad\textbf{(B)}~\frac{5\sqrt{3}}{11}\qquad\textbf{(C)}~\frac{4}{5}\qquad\textbf{(D)}~\frac{11}{13}\qquad\textbf{(E)}~\frac{7\sqrt{3}}{13}</math> | ||
− | ==Solution | + | ==Solution 1== |
− | + | Let O be circumcenter of the equilateral triangle | |
− | + | Easily get <math>OF = \frac{14\sqrt{3}}{3}</math> | |
− | 2 | + | <math>2 \cdot \triangle(OFC) + \triangle(OCE) = OF^2 \cdot \sin(\theta) + OF^2 \cdot \sin(120 - \theta)</math> |
− | + | <cmath> = \frac{14^2 \cdot 3}{9} ( \sin(\theta) + \sin(120 - \theta) ) </cmath> | |
− | + | <cmath> = \frac{196}{3} ( \sin(\theta) + \sin(120 - \theta) ) </cmath> | |
− | + | <cmath> = 2 \cdot {\frac{1}{3} } \cdot(ABCDEF) = 2\cdot \frac{91\sqrt{3}}{3} </cmath> | |
− | |||
− | |||
− | |||
− | |||
− | |||
− | |||
− | |||
− | |||
− | |||
− | |||
− | |||
− | |||
+ | <cmath> \sin(\theta) + \sin(120 - \theta) = \frac{13\sqrt{3}}{14} </cmath> | ||
+ | <cmath> \sin(\theta) + \frac{ \sqrt{3}}{2}\cos( \theta) +\frac{ \sqrt{1}}{2}\sin( \theta) = \frac{13\sqrt{3}}{14} </cmath> | ||
+ | <cmath> \sqrt{3} \sin( \theta) + \cos( \theta) = \frac{13 }{7} </cmath> | ||
+ | <cmath> \cos( \theta) = \frac{13 }{7} - \sqrt{3} \sin( \theta) </cmath> | ||
+ | <cmath> \frac{169 }{49} - \frac{26\sqrt{3} }{7} \sin( \theta) + 4 \sin( \theta)^2 = 1 </cmath> | ||
+ | <cmath> \sin( \theta) = \frac{5\sqrt{3} }{14} or \frac{4\sqrt{3} }{7} </cmath> | ||
+ | <math>\frac{4\sqrt{3} }{7} </math> is invalid given <math>\theta \leq 60^\circ </math> , <math>\sin(\theta ) < \sin( 60^\circ ) = \frac{\sqrt{3} }{2} = \frac{\sqrt{3} \cdot 3.5}{7} </math> | ||
+ | <cmath> \cos( \theta) = \frac{11 }{14} </cmath> | ||
+ | <cmath> \tan( \theta) = \frac{5\sqrt{3} }{11} \boxed{B } </cmath> | ||
~[https://artofproblemsolving.com/wiki/index.php/User:Cyantist luckuso] | ~[https://artofproblemsolving.com/wiki/index.php/User:Cyantist luckuso] | ||
Line 38: | Line 35: | ||
==Solution #2== | ==Solution #2== | ||
From <math>\triangle ABC</math>'s side lengths of 14, we get OF = OC = OE =<cmath>\frac{14\sqrt{3}}{3} </cmath> | From <math>\triangle ABC</math>'s side lengths of 14, we get OF = OC = OE =<cmath>\frac{14\sqrt{3}}{3} </cmath> | ||
− | We let angle FOC = (\theta) | + | We let angle FOC = (<math>\theta</math>) |
+ | And therefore angle EOC = 120 - (<math>\theta</math>) | ||
The answer would be 3 * (Area <math>\triangle FOC</math> + Area <math>\triangle COE</math>) | The answer would be 3 * (Area <math>\triangle FOC</math> + Area <math>\triangle COE</math>) | ||
− | Which area <math>\triangle FOC</math> = 0.5 * <math>\frac{14\sqrt{3}}{3} | + | Which area <math>\triangle FOC</math> = 0.5 * <math> (\frac{14\sqrt{3}}{3} )^2 * sin(\theta)</math> |
− | And area <math>\triangle COE</math> = 0.5 * <math>\frac{14\sqrt{3}}{3} | + | And area <math>\triangle COE</math> = 0.5 * <math> (\frac{14\sqrt{3}}{3} )^2 * sin(120 - \theta)</math> |
− | Therefore the answer would be 3 * 0.5 * (<math>\frac{14\sqrt{3}}{3} | + | Therefore the answer would be |
+ | 3 * 0.5 * (<math>\frac{14\sqrt{3}}{3} )^2 * (sin(\theta)+sin(120 - \theta)) = {91\sqrt{3}}</math> | ||
Which <cmath> sin(\theta)+sin(120 - \theta) = \frac{91\sqrt{3}}{98} </cmath> | Which <cmath> sin(\theta)+sin(120 - \theta) = \frac{91\sqrt{3}}{98} </cmath> | ||
− | So <cmath> \frac{1}{2}cos(\theta)+\frac{\sqrt{3}}{2} = \frac{91}{98} </cmath> | + | So <cmath> \frac{1}{2}cos(\theta)+\frac{\sqrt{3}}{2}sin(\theta) = \frac{91}{98} </cmath> |
Therefore <cmath> sin(\theta + 30) = \frac{91}{98} </cmath> | Therefore <cmath> sin(\theta + 30) = \frac{91}{98} </cmath> | ||
Line 58: | Line 57: | ||
Which <cmath> tan(\theta + 30) = \frac{91}{21\sqrt{3}} = \frac{91\sqrt{3}}{63} </cmath> | Which <cmath> tan(\theta + 30) = \frac{91}{21\sqrt{3}} = \frac{91\sqrt{3}}{63} </cmath> | ||
− | < | + | <math>tan(\theta)</math> can be calculated using addition identity, which gives the answer of |
− | |||
− | |||
+ | <cmath>(B)\frac{5\sqrt{3}}{11}</cmath> | ||
+ | (I would really appreciate if someone can help me fix my code and format) | ||
+ | ~mitsuihisashi14 | ||
+ | ~[https://artofproblemsolving.com/wiki/index.php/User:Cyantist luckuso] (fixed Latex error ) | ||
+ | ==Video Solution by SpreadTheMathLove== | ||
+ | https://www.youtube.com/watch?v=akLlCXKtXnk | ||
==See also== | ==See also== | ||
{{AMC12 box|year=2024|ab=B|num-b=18|num-a=20}} | {{AMC12 box|year=2024|ab=B|num-b=18|num-a=20}} | ||
{{MAA Notice}} | {{MAA Notice}} |
Latest revision as of 14:54, 19 November 2024
Problem 19
Equilateral with side length is rotated about its center by angle , where , to form . See the figure. The area of hexagon is . What is ?
Solution 1
Let O be circumcenter of the equilateral triangle
Easily get
is invalid given ,
Solution #2
From 's side lengths of 14, we get OF = OC = OE = We let angle FOC = () And therefore angle EOC = 120 - ()
The answer would be 3 * (Area + Area )
Which area = 0.5 *
And area = 0.5 *
Therefore the answer would be 3 * 0.5 * (
Which
So
Therefore
And
Which
can be calculated using addition identity, which gives the answer of
(I would really appreciate if someone can help me fix my code and format)
~mitsuihisashi14 ~luckuso (fixed Latex error )
Video Solution by SpreadTheMathLove
https://www.youtube.com/watch?v=akLlCXKtXnk
See also
2024 AMC 12B (Problems • Answer Key • Resources) | |
Preceded by Problem 18 |
Followed by Problem 20 |
1 • 2 • 3 • 4 • 5 • 6 • 7 • 8 • 9 • 10 • 11 • 12 • 13 • 14 • 15 • 16 • 17 • 18 • 19 • 20 • 21 • 22 • 23 • 24 • 25 | |
All AMC 12 Problems and Solutions |
The problems on this page are copyrighted by the Mathematical Association of America's American Mathematics Competitions.